LSAT and Law School Admissions Forum

Get expert LSAT preparation and law school admissions advice from PowerScore Test Preparation.

 Administrator
PowerScore Staff
  • PowerScore Staff
  • Posts: 8917
  • Joined: Feb 02, 2011
|
#38149
Complete Question Explanation
(The complete setup for this game can be found here: lsat/viewtopic.php?t=14969)

The correct answer choice is (B)

If H is evaluated by R, it follows that K must be evaluated by T, who must consequently evaluate only one applicant:
PT71_D13 LG Explanations_Game #2_#7_diagram 1.png
Since S must evaluate more applicants than T, it would be wise to focus our attention on S. After accounting for H, K, and G, there are only four applicants remaining (F, I, L, and M), at least two of whom must be evaluated by S. However, recall that I and M cannot be evaluated by the same officer (third rule) whereas F and L must be evaluated by the same officer (second rule). Clearly, if S did not evaluate F and L, then S would have at most one applicant to evaluate (M or I, but not both), in direct violation of the last rule of the game. Therefore, F and L must be evaluated by S:
PT71_D13 LG Explanations_Game #2_#7_diagram 2.png
The assignment of the remaining two variables—I and M—is unknown. Because you are solving a Could Be True question, you should expect that the correct answer choice will exploit this uncertainty. Only answer choices (B) and (E) address the placement of I and M, and only answer choice (B) could be true. Answer choice (E) cannot be true, because T evaluates K, and K only. Answer choices (A) and (D) are both incorrect, because F and L must be evaluated by S. Answer choice (C) is also incorrect, because K is evaluated by T.
You do not have the required permissions to view the files attached to this post.
 medialaw111516
  • Posts: 80
  • Joined: Dec 11, 2018
|
#71902
Hi, I got this one right by running through all the options until I got a could be true (luckily it was only B), but I'm wondering if there's another way to do these? I usually start by looking at my most limiting variables so here I placed the K on T and then decided I needed to focus on filling up the S since there was that rule about S having more than T, so that drew me to try answer B first because S was in in the answer, but I'm not sure if that's a great strategy.
 Claire Horan
PowerScore Staff
  • PowerScore Staff
  • Posts: 408
  • Joined: Apr 18, 2016
|
#71919
Hi MediaLaw111516,

It might have been faster for you to approach this question through process of elimination. Once you add H to R's space on your diagram, you can infer that K must be evaluated by T and T can't evaluate anyone else. You also know that F and L must be evaluated by S because otherwise I and M are the only ones left for S, and they cannot go together. This information allows you to eliminate every answer but the right answer. For could-be-true questions, it is often fastest to figure out what must be false, eliminate those choices, and then only have a couple answers left to test. In this case, you would have very quickly arrived at the only possible answer, B.

Good luck with your studies!

Get the most out of your LSAT Prep Plus subscription.

Analyze and track your performance with our Testing and Analytics Package.